LSAT and Law School Admissions Forum

Get expert LSAT preparation and law school admissions advice from PowerScore Test Preparation.

 Administrator
PowerScore Staff
  • PowerScore Staff
  • Posts: 8919
  • Joined: Feb 02, 2011
|
#24683
Complete Question Explanation

Strengthen—PR. The correct answer choice is (B)

In this question, the stimulus is fairly straightforward, while the question stem might take some time to understand. The author of the stimulus tells us that a proposed law would require a seven day waiting period for hand gun purchase, intended to allow for checking of prison records to avoid putting guns in the hands of likely criminals. Critics oppose the law, based on the fact that the records would also prohibit many law abiding citizens from getting guns.

The question stem appears to have been written in a deliberately convoluted way, but basically, the question is as follows:

If it’s true, as critics say, that law abiding citizens would also be prevented from buying guns, which of these principles would strengthen the argument against the new law?

The correct answer choice will likely present a principle which, when applied, would prioritize the rights of law-abiding citizens to buy guns, over the importance of the recently proposed precautions.

Answer choice (A): The author does not weight the rights of citizens against the rights of criminals, but rather the rights of citizens against the value of the proposed precautionary measures.

Answer choice (B): This is the correct answer choice. If this principle is applied, then the rights of citizens should not be restricted in the name of extra precaution. This would clearly strengthen the case of the opposition.

Answer choice (C): The opposition is not arguing that no benefit would come from the new legislation, but that the benefits do not justify restricting the rights of law-abiding citizens.

Answer choice (D): This principle would not strengthen the opposition argument, because this principle would allow for the new law to place the proposed restrictions.

Answer choice (E): Adequate training is not discussed in the stimulus, so this principle does not strengthen the opposition argument, and this answer choice is incorrect.
 yournoona
  • Posts: 18
  • Joined: Mar 13, 2020
|
#74707
Hello.
Although I got the answer to the question as B. I have problem understading the acronyms used throughout.
Could you please tell me the full forms of
SN
FL
PR
X
AP
#%
I can't seem to find any full forms relating to this in the GMAT critical reasoning bible as well.
Thankyou.
 Adam Tyson
PowerScore Staff
  • PowerScore Staff
  • Posts: 5153
  • Joined: Apr 14, 2011
|
#74739
You bet - here's a quick glossary!

SN = Sufficient and Necessary, an indication that Conditional Reasoning is present
FL = Formal Logic, arguments based on "some" and "most" for the most part
PR = Principle, an indication that the question involves applying or identifying a rule or guideline
X = Except, a question stem that identifies the four wrong answers and wants you to pick the one that doesn't fit
AP = Argument Part (Method-AP), which means a question that asks you to identify the role that a particular claim plays in an argument
#% = Numbers and Percentages, indicating a question that involves those concepts and might involve some mathematical analysis (averages, ratios, rates, frequency, etc.)

Get the most out of your LSAT Prep Plus subscription.

Analyze and track your performance with our Testing and Analytics Package.